¿Qué conductancia se mide para el estado de Hall de espín cuántico cuando la conductancia de Hall desaparece?

Probablemente sea solo una definición, pero ¿qué hicieron König et al. medir realmente cuando confirmó la existencia de estados superficiales en pozos cuánticos CdTe/HgTe/CdTe (ver http://arxiv.org/abs/0710.0582 )?

Según la mayoría de las revisiones (p. ej ., http://www.annualreviews.org/doi/pdf/10.1146/annurev-conmatphys-062910-140538 ):

"Sin embargo, debido a que la conductancia Hall del estado QSH se desvanece, está claro que el número TKNN o Chern discutido anteriormente, que corresponde al valor de la conductancia Hall en unidades de e2/h, no puede proporcionar una clasificación útil del QSH. Este problema se ha abordado tanto en la teoría de la banda topológica (23) como en la teoría del campo topológico (23). En consecuencia, el invariante topológico adecuado se valora en el grupo Z2 que contiene solo dos elementos, 0 o 1, con 1 correspondiente a el aislador QSH topológicamente no trivial y 0 correspondiente a un aislador topológicamente trivial sin estados de borde robustos sin espacios ".

y

"Debido a que el acoplamiento espín-órbita destruye la conservación del espín, no existe una conductancia SH cuantificada en el efecto QSH. Esta es otra forma de entender por qué la invariante topológica correcta para el efecto QSH es Z2 y no Z. Finalmente, el BHZ Hamiltonian predice un solo estado de borde helicoidal por borde".

no entiendo esto ¿Entonces no hay conductancia de carga, pero medimos la conductancia de carga? ¿Cuál es la diferencia entre la conductancia de espín y de carga? Pensé que König midió una conductancia de carga que era exactamente el doble de la conductancia de Hall (e ^ 2 / h) (para mí, eso está cuantificado ...). ¿Significa esto que en realidad solo hay dos estados que conducen al doble de la conductancia de Hall? Además: ¿Por qué solo hay un solo estado de borde helicoidal por borde? ¿Por qué debemos tener al menos uno y por qué no podemos tener, digamos, dos estados por borde?

Respuestas (1)

Probablemente sea solo una definición, pero ¿qué hicieron König et al. realmente midió cuando confirmó la existencia de estados superficiales en pozos cuánticos de CdTe/HgTe/CdTe (ver http://arxiv.org/abs/0710.0582 ).... ...Así que no hay conductancia de carga, pero medimos la carga ¿conductancia? ¿Cuál es la diferencia entre la conductancia de espín y de carga? Pensé que König midió una conductancia de carga que era exactamente el doble de la conductancia de Hall ( mi 2 / h ) (Para mí eso está cuantizado...).?

Sí, König et al. de hecho, midió la conductancia de carga en pozos cuánticos de CdTe/HgTe/CdTe. Creo que su dilema es el resultado de mezclar la descripción de las propiedades del aislador Hall de espín cuántico con y sin la existencia de polarización externa . La imagen intuitiva de estados de borde que se propagan en sentido contrario con giros opuestos, que se discute repetidamente en la literatura, no tiene sesgos externos. Imagine una capa 2D de HgTe (en el régimen invertido) simplemente sentada allí sin que nadie le haga nada. Centrándose en (digamos) el borde superior, tiene (digamos) | k , propagándose a la derecha (con conductancia mi 2 / h ) y es el socio de Kramer | k , propagando la izquierda. En ausencia de un sesgo externo, los niveles de Fermi de ambos estados son iguales. Por eso la conductancia de la carga es σ cobrar = mi 2 / h + ( mi 2 / h ) = 0 (donde el signo menos proviene del hecho de que la corriente del | k , estado fluye en la dirección opuesta). Sin embargo, una corriente de espín que va en una dirección (digamos + X ^ ) y la misma magnitud de corriente de giro descendente que va en la otra dirección ( X ^ ) es equivalente al doble de la cantidad de corriente de giro en el + X ^ dirección. Es por eso que terminas con una conductancia de espín de 2 mi 2 / h .

Ahora, en el estudio de König et al. experimento de transporte las corrientes de carga debidas a | k parte superior , y | k parte superior , no se anulan perfectamente. En otras palabras (digamos) en el borde superior el nivel cuasi-Fermi de | k , es mayor que el nivel cuasi-Fermi de | k , . Esta diferencia en los niveles de Fermi correspondería a un flujo neto de electrones en el + X ^ dirección a lo largo del borde superior. Este flujo neto da lugar a una conductancia de mi 2 / h . En el borde inferior, sin embargo, cuasi-Fermi nivel de | k abajo , es mayor que el nivel cuasi-Fermi de | k abajo , . Por lo tanto, nuevamente tiene un flujo neto de electrones en el + X ^ dirección a lo largo del borde inferior. Esto da lugar a otro canal con conductancia mi 2 / h . Por lo tanto, la contribución total sería 2 mi 2 / h . Lo que acabo de describir se aplica a una medición de resistencia (o conductancia) de dos terminales. Si paso una corriente yo entre los dos contactos entonces el voltaje (proporcional a la diferencia en los niveles de Fermi en los dos contactos) sería V = ( h / 2 mi 2 ) yo . Una forma de cuantificar este análisis es utilizando la fórmula de Landauer-Büttiker

yo i = mi h j ( T j i m i T i j m j )
donde las cantidades con índice de subíndice único indican a qué contacto pertenecen estas cantidades. Por ejemplo, considere un dispositivo de seis terminales de la siguiente manera

ingrese la descripción de la imagen aquí

Puede observar la contrapropagación del giro hacia arriba (por ejemplo, rojo) y el giro hacia abajo (azul) a lo largo de los bordes superior e inferior. La cantidad T i j representa la probabilidad de transmisión para que el electrón pase del contacto i j . Como se puede observar en la figura solamente T i , i + 1 y T i + 1 , i será distinto de cero. Como la teoría del efecto Hall de espín cuántico predice que los estados de borde son robustos al desorden (no magnético), debemos tener

T i , i + 1 = T i + 1 , i = 1
es decir, transmisión perfecta (sin disipación). Reemplazando esto en la fórmula de Landauer-Büttiker (y asumiendo una corriente que pasa del contacto 1 4 ) arriba obtendrá seis ecuaciones lineales en seis incógnitas m i :
mi h ( 2 1 0 0 0 1 1 2 1 0 0 0 0 1 2 1 0 0 0 0 1 2 1 0 0 0 1 1 2 1 1 0 0 0 1 2 ) A ( m 1 m 2 m 3 m 4 m 5 m 6 ) X = yo 14 ( 1 0 0 1 0 0 ) b .

Sin embargo, hay una redundancia en este sistema de ecuaciones (o det ( A ) = 0 ). No todos los m i son realmente desconocidos. podemos establecer m 4 = 0 (es decir, potencial de referencia o tierra). En ese caso, puede reducir el sistema de ecuaciones a

mi h ( 2 1 0 0 1 1 2 1 0 0 0 1 2 0 0 0 0 1 1 0 1 0 0 1 2 ) ( m 1 m 2 m 3 m 5 m 6 ) = yo 14 ( 1 0 0 1 0 ) .
Ahora, resolviendo esto obtienes
( m 1 m 2 m 3 m 5 m 6 ) = yo 14 h mi ( 2 1 0 0 1 1 2 1 0 0 0 1 2 0 0 0 0 1 1 0 1 0 0 1 2 ) 1 ( 1 0 0 1 0 ) ,
( m 1 m 2 m 3 m 5 m 6 ) = yo 14 h mi ( 3 / 2 1 1 / 2 1 / 2 1 ) .
Diferencia de voltaje entre contactos i y j es
V i j = 1 ( mi ) ( m i m j ) .
Puedes comprobar eso
V 14 = ( 3 h 2 mi 2 ) yo 14
y
V 23 = ( h 2 mi 2 ) yo 14 .

Esto es exactamente lo que Roth et. al observado experimentalmente

ingrese la descripción de la imagen aquí

Se confirmó que estos valores de resistencia de dos y cuatro terminales son independientes de la geometría de la muestra ( L y W ). Como resultado, puede descartar cualquier tipo de conducción que no sean los estados de borde. Además, espera estos valores de resistencia solo si asume estados de borde helicoidales y sin disipación . Por lo tanto, estas medidas deberían confirmar la existencia del estado Hall de espín cuántico en HgTe.

Además: ¿Por qué solo hay un solo estado de borde helicoidal por borde? ¿Por qué debemos tener al menos uno y por qué no podemos tener, digamos, dos estados por borde?

Si tuviera (digamos) dos pares de estados de borde helicoidales que se propagan en sentido contrario, dicho sistema no es resistente al desorden. Tal situación se muestra en la parte (a) (en ausencia de desorden) en la siguiente figura. Los estados pertenecientes a las bandas roja y azul a la misma altura (es decir, misma energía mi ( k , ) = mi ( k , ) ) forman los socios de Kramers. Las regiones sombreadas en marrón representan las bandas a granel. Dado que tiene dos pares de socios de Kramers, las bandas respectivas se intersecarán naturalmente en dos puntos (como se muestra en la parte (a)). Cualquier tipo de desorden dará como resultado la salida de los estados como se muestra en la parte (b). Pero tenga en cuenta que tal proceso de separación está permitido por el teorema de Kramers. Una forma rápida de ver esto es: observe el reflejo de cualquier banda con respecto a la vertical ( k = 0 ) eje. Bajo tales reflejos, el rojo debería transformarse en azul y viceversa.

ingrese la descripción de la imagen aquí

Ahora, imagine que tiene dos copias de estados de borde helicoidales similares a Dirac. En otras palabras, dos copias de la parte (d) superpuestas una sobre la otra. Cuando abra el sistema, se verá como la parte (c). Puede observar que en la parte (c) la simetría de inversión de tiempo aún se conserva después de la separación. En la parte (d), sin embargo, solo tiene una copia de los socios de Kramers. Solo hay un punto de intersección (a diferencia de las partes (a) y (b)). Puedes observar que no hay manera de que puedas abrir una brecha (en k = 0 ) sin dejar de satisfacer la simetría de inversión de tiempo. Más específicamente, la introducción de una brecha (en la parte (d)) solo violará el teorema de Kramers en el k = 0 punto (es decir mi ( k , ) y mi ( k , ) no será igual en k = 0 ). Por lo tanto, si el desorden respeta la simetría de inversión de tiempo, se dice que dicha intersección de bandas está "protegida por la simetría de inversión de tiempo". En sistemas realistas como el pozo cuántico HgTe, supongamos que tenía 2 norte + 1 Socios de Kramers. En ese caso, el desorden destruirá a los socios de tales Kramers en norte pares tales que solo quede un par al final. La existencia de un número impar de pares está garantizada en una fase topológicamente no trivial. De hecho, así es como la gente identifica una fase topológicamente no trivial.

Debido a que el acoplamiento espín-órbita destruye la conservación del espín, no existe una conductancia SH cuantificada en el efecto QSH. Esta es otra forma de entender por qué la invariante topológica correcta para el efecto QSH es Z 2 y no Z . Finalmente, el hamiltoniano BHZ predice un solo estado de borde helicoidal por borde.

Debes leer las líneas anteriores a las anteriores. Los autores mencionaron que el espín no es un buen número cuántico. Cuando introduce el acoplamiento de la órbita de espín, el hamiltoniano es diagonal en la base del momento angular total . El momento angular total se puede definir como

j ^ = L ^ + S ^
y puede etiquetar los estados propios como | j , metro j , s dónde j ( j + 1 ) , metro j , y s son los valores propios de j ^ 2 , j ^ z , y S ^ z . En la mayor parte de HgTe metro j es un buen número cuántico en lugar de s . En el borde, sin embargo, incluso metro j no se conserva debido a la falta de simetría rotacional. Creo que lo que los autores están tratando de hacer es enfatizar la diferencia entre el efecto Hall de espín cuantizado y el efecto Hall de espín cuántico. Como describiré en breve, el efecto Hall de espín cuantizado no es posible. Por ejemplo, supongamos que está tratando de observar el giro análogo del efecto Hall cuántico entero. Pasa una corriente longitudinal en un material ferromagnético (donde se conserva el giro), luego obtendría los mismos pasos enteros Z . Habrá acumulación tanto de espín como de carga en la dirección transversal. Heurísticamente, esto es como un híbrido entre los efectos Hall de giro y carga. Esto también se conoce como el efecto Hall anómalo cuántico. Pero tenga en cuenta que la simetría de inversión de tiempo se rompe en tal sistema. En el momento en que introduces la simetría de inversión del tiempo Z se derrumba a Z 2 . Si tuviera un efecto Hall de giro puro , es decir, hay giro pero no acumulación de carga en la dirección transversal, entonces se conserva la simetría de inversión de tiempo. Por lo tanto, nunca observará un valor cuantificado (o Z ) efecto Hall de espín.

Eche un vistazo a esta sugerencia de edición, que he rechazado porque se desvía de la intención original de los autores de la publicación. Sin embargo, si la corrección tiene sentido para usted, haga lo necesario, incorporándola. Salud :)
yo i representa el flujo de corriente hacia el terminal i , ¿Correcto?
Sí, dentro o fuera, dependiendo de tu convención.
Creo que hay un error en la ecuación de las corrientes después de poner a tierra el terminal 4, de hecho el término con -1 debería desaparecer, si no no es posible recuperar los resultados de Roth el. al .
@DarioBercioux: Ec. (4) está bien. Sin embargo, la transición de (3) a (4) podría haber sido engañosa. No hay una manipulación matricial trivial para pasar de (3) a (4). Escribí explícitamente el sistema de ecuaciones en (3), conjunto m 4 = 0 , reorganizó el sistema de ecuaciones y reconstruyó la ecuación matricial en (4).
@NanoPhys lo que no está claro en su publicación útil es cómo obtener la manipulación de matriz no trivial para pasar de (3) a (4). Porque en realidad, cuando dices que pones a tierra el terminal 4, sería suficiente eliminar la cuarta línea y la columna de la matriz de conexión, y el cuarto elemento del vector de la corriente. De esta forma se obtiene el mismo resultado pero sin realizar la manipulación de la matriz. En realidad, los dos enfoques son equivalentes. Gracias por responder.
@DarioBercioux: Tienes razón, no es del todo obvio que me salté un montón de pasos de (3) a (4); por lo tanto, fue engañoso! Y, por cierto, simplemente eliminar la cuarta fila y columna de la matriz LHS, y el "-1" del vector RHS, no funcionará. Si hace eso, el vector de solución que obtendrá es (-9/5, -6/5, -3/5, -1, -7/5). Que "-1" esté en el RHS es fundamental. Tenga en cuenta que cuando expande la cuarta ecuación lineal en (3) y establece m 4 = 0 , obtienes la condición m 5 + m 6 = 1 . Entonces el "-1" todavía sobrevive en el RHS de (4).
@NanoPhys Puedo seguirte parcialmente porque no conozco los detalles de la rotación que has realizado. Sin embargo, siguiendo el planteamiento que puedes encontrar en el Cap. 4 del libro de Datta "Transporte electrónico en sistemas mesoscópicos", eliminando la cuarta línea y la columna y teniendo solo la corriente yo 1 , recupero exactamente los mismos resultados que tú para V 14 y V 23 . Puedo enviarte alguna nota si quieres.